.
. centre de ressources dilemmes et doutes le visage humain de mathématiques Qui sommes-nous Problème de mois activités de promotion babillard
Centrale des maths - centraledesmaths.uregina.ca
Problème du mois
Problème
du mois
  Problèmes récents
et solutions
Anciens problèmes
2005/2006 06/07 07/08 08/09 09/10 10/11 11/12

Solution au problème de février 2011

Le problème:
.

Déterminer les valeurs des nombres naturels $n$ et $a_1, a_2, a_3, \dots, a_n$ telles que $a_1 + a_2 + a_3 + \dots + a_n = 2011$ et le produit $a_1\cdot a_2 \cdot a_3 \cdot \dots \cdot a_n$ est le plus grand possible.

Nous avons reçu les solutions correctes de

Lou Cairoli (États Unis)

Bernard Collignon (France)

Mei-Hui Fang (Autriche)

Philippe Fondanaiche (France)

Bruce Golfman (Autriche)

Gruian Cornel (Roumanie)

Verena Haider (Autriche)

Benoît Humbert (France)

Ile Ilijevski (Macédoine)

Kipp Johnson (États Unis)

Wolfgang Kais (Allemagne)

Normand LaLiberté (Ontario)

Matthew Lim (États Unis)

Patrick J. LoPresti (États Unis)

John T. Robinson (États Unis)

Mathias Schenker (Suisse)

Heri Setiyawan (Indonésie)

Albert Stadler (Suisse)

Paul Voyer (France)


La solution:

Nous reproduisons la solution de Humbert et la généralisation de Collignon.

Solution de Humbert: "Soient n nombres naturels $a_1, a_2, a_3, \dots, a_n$ tels que $a_1 + a_2 + a_3 + \dots + a_n = 2011$

  • Si l'un des $a_i$ est nul, alors le produit $a_1, a_2, a_3, \dots, a_n$ est nul, et n'est pas le plus grand possible.

  • Si l'un des $a_i$ est égal à 1, on peut le supprimer et ajouter 1 à l'un des autres facteurs $a_j$ et obtenir ainsi un produit strictement plus grand.

  • Si l'un des $a_i$est supérieur ou égal à 5, on peut le décomposer en 2 et $a_i - 2$ et obtenir ainsi un produit strictement plus grand car $ 2 \times (a_i - 2) = a_i + (a_i - 4) > a_i.$

  • Si l'un des $a_i$ est égal à 4, on peut le décomposer en 2 et 2 sans changer la valeur du produit : 2×2 = 4 .

  • Si trois facteurs $a_i, a_j, a_k$ sont égaux à 2, on peut les remplacer par deux facteurs 3 (car $2 + 2 + 2 = 3 + 3$ ) et obtenir ainsi un produit strictement plus grand car $3 \times 3 > 2 \times 2 \times 2.$

Ainsi, les deux solutions du problème sont :

Solution 1 : $n = 671, a_1 = \dots = a_{669} = 3 \mbox{ et } a_{670} = $ $a_{671} = 2.$

Solution 2 : $n = 670, a_1 = ... = a_{669} = 3 \mbox{ et } a_{670} = 4.$''

La valeur maximale du produit, calculée par Cairoli et Jim Altieri, est 625430993058475627735345593858714553015081394708637260475756313800
148211843840935329415403222795915589302569939205425897003519805509
694939731310493989960203598356927645249925489567206575014415623015
699165339810785069996964459321850612813854978894045855958144172599
78814911519620863137698556067203931868434684932884205132.

Bernard Collignon s'est intéressé au cas de produits de réels positifs plutôt que d'entiers: Il utilise le lemme bien connu

Lemme 1 : Pour tous nombres réels positifs x et y de somme S, le produit maximum P = x × y est atteint lorsque x = y.

et sa généralisation naturelle à plusieurs termes:

Lemme 2 : Pour tout entier positif n > 1 , le produit de n nombres réels positifs $x_1, \dots, x_n$ de somme S est maximum lorsque tous ces nombres réels sont égaux.

Et détermine ainsi que tous les facteurs doivent valoir $\frac{2011}{n}.$

"l nous reste donc à déterminer quelle valeur donner à n ( le nombre de facteurs ) pour obtenir le produit recherché X le plus grand possible. On doit donc optimiser le produit : $\left( \frac{2011}{n}\right)^n$ où $n \in IN^*$. Pour cela, on considère la fonction f définie sur $IR^{+*}$ par :

\[ f(x) = \left( \frac{2011}{x}\right)^x \]

Dérivée : $f(x) = \exp\left(x \ln\frac{2011}{x} \right) = \exp(u(x))$ du type $f = e^u,$ avec

\[ u(x) = \ln (f (x)) = x \ln\frac{2011}{x} = x \ln(2011) – x \ln(x). \]

Donc $f’ = u’ \exp(u) \mbox{ et } f’(x)$ est donc du signe de $u’(x).$

On a : $u’ (x) = \ln (2011) – ( \ln (x) + \frac{x}{x}) = \ln \frac{2011}{e} – \ln(x).$

Variations :

x

0

1

p

$\frac{2011}{e}$

p+1


+∞

Signe de u’(x) = signe de f’(x)


+


0


-


Variation de f(x)

1

croissante

$M_1$

M

$M_2$

décroissante

0

Le maximum de f est obtenu lorsque $x = \frac{2011}{e}$ qui n’est pas un nombre entier car e est irrationnel. Comme le nombre de facteurs est forcément entier, la solution X à notre problème est soit $M_1$ soit $M_2$, nombres qui correspondent respectivement à un nombre p ou p+1 de facteurs égaux de somme 2011, avec $p = \left[\frac{2011}{e}\right],$ c'est-à-dire que p est la partie entière de $\frac{2011}{e.}$

Comme f et u = ln f ont les mêmes variations (ce qu’on peut utiliser plus tôt), il est équivalent de comparer les logarithmes de $M_1$ et $M_2.$

Tout d’abord : $\frac{2011}{e} \approx 739,8$ donc $p = \left[\frac{2011}{2}\right] = 739 \mbox{ et } p+1 = 740.$

$\ln M_1= u (739) = 739 \ln \left(\frac{2011}{739}\right) \approx 739, 80512$

$\ln M_2= u (740) = 740 \ln \left(\frac{2011}{740} \right) \approx 739, 80553.$

Donc il faut prendre : $n = 740$ et pour tout $k \in {1, ..., 740}$ : $a_k= \frac{2011}{740}.$

On obtient alors le plus grand produit X possible qui est : $X = f (740) = \left(\frac{2011}{740}\right)^{740}.$''

 

 

 


Centrale des maths reçoit une aide financière de l’Université de Regina et de The Pacific Institute for the Mathematical Sciences.

CMS
.

 

accueil centre de ressources accueil Société mathématique du Canada l'Université de Regina PIMS